a Fill in the blank. If necessary, use the slash mark (/) for a fraction bar. If cosg = then tang =

A Fill In The Blank. If Necessary, Use The Slash Mark (/) For A Fraction Bar. If Cosg = Then Tang =

Answers

Answer 1
Explanation

We can use a right triangle and the below trigonometric ratios.

[tex]\begin{gathered} \cos(\theta)=\frac{\text{ Adjacent leg}}{\text{ Hypotenuse}} \\ \tan(\theta)=\frac{\text{ Opposite leg}}{\text{ Adjacent leg}} \end{gathered}[/tex]

In this case, we have:

[tex]\cos(\theta)=\frac{3}{5}=\frac{\text{Adjacent leg}}{\text{Hypotenuse}}[/tex]

As we can see, we need to know the value of the opposite leg. Since it is a right triangle, we can use the Pythagorean theorem formula.

[tex]\begin{gathered} a^2+b^2=c^2 \\ \text{ Where} \\ a\text{ and }b\text{ are the legs} \\ c\text{ is the hypotenuse} \end{gathered}[/tex]

Then, we have:

[tex]\begin{gathered} a=3 \\ b=? \\ c=5 \\ a^{2}+b^{2}=c^{2} \\ 3^2+b^2=5^2 \\ 9+b^2=25 \\ \text{ Subtract 9 from both sides} \\ 9+b^2-9=25-9 \\ b^2=16 \\ $$\text{ Apply square root to both sides of the equation}$$ \\ \sqrt{b^2}=\sqrt{16} \\ b=4 \end{gathered}[/tex]

Finally, we have:

Then, we can find the value of tan(θ):

[tex]\begin{gathered} \tan(\theta)=\frac{\text{Opposite leg}}{\text{Adjacentleg}} \\ \tan(\theta)=\frac{4}{3} \end{gathered}[/tex]Answer[tex]\tan(\theta)=\frac{4}{3}[/tex]

A Fill In The Blank. If Necessary, Use The Slash Mark (/) For A Fraction Bar. If Cosg = Then Tang =
A Fill In The Blank. If Necessary, Use The Slash Mark (/) For A Fraction Bar. If Cosg = Then Tang =

Related Questions

Given g(x) = 1/x^3Explain if the question cannot be solved

Answers

Given

[tex]g(x)=\frac{1}{x^3}[/tex]

To find:

[tex]\int_{-\frac{\pi}{2}}^{\frac{\pi}{2}}g(x)dx[/tex]

Explanation:

It is given that,

[tex]g(x)=\frac{1}{x^3}[/tex]

That implies,

[tex]\int_{-\frac{\pi}{2}}^{\frac{\pi}{2}}g(x)dx[/tex]

The office manager orders computer paper every 6 weeks and printer ink cartridges every 8 weeks.

QUESTION: If she places an order for both paper and ink this week, how many weeks will it be until she orders them both in the same week again?

Answers

The number of weeks that she orders them both in the same week again is 24 weeks.

How to illustrate the information?

From the information, the office manager orders computer paper every 6 weeks and printer ink cartridges every 8 weeks.

The weeks when they will order same will be the least common multiple for 6 and 8. This will be:

6 = 6, 12, 18, 24

8 = 8, 16, 24

The multiple is 24 weeks. This illustrates the information.

Learn more about multiples on:

brainly.com/question/26856218

#SPJ1

choose the equation that could be used to find two consecutive integers whose sum is 67.

Answers

d) n + (n +1) = 67

1) Let's call the first number as n, and its consecutive as n +1

Then we can write:

n + (n +1) = 67

n +n +1 = 67

2n +1 = 67

2n = 66

n= 33

and n+1 = 34

2) Hence, the answer is n + (n +1) = 67

how to solve 4|x|+|-4|=|-6|

Answers

x = 1/2, x = -1/2

Simplify:

4|x| + |-4| = |-6|

4|x| + 4 = 6

4|x| = 2

|x| = 1/2

Solutions:

1) x = 1/2

2) x = -1/2

Which of the following is equivalent to tanблOA. tan 3OB. tan 5OC. tanOD. tanВп3Reset Selection

Answers

Okay, here we have this:

Considering the provided expression, we are going to identify to which is equivalent, so we obtain the following:

We obtain that the correct answer is the option C, because:

two students determined the value of this expression

these are the steps each student used




analyze the steps and describe any eras made by student p and student q

Answers

Answer:

Student P made a mistake at step 1

Student Q made a mistake at step 3

Explanation:

First, let's analyze what are the correct steps to find the value of the expression.

So, the first step is to apply the distributive property:

[tex]\begin{gathered} -2.5(1.4+3.1)+6.9(-4.3) \\ -2.5(1.4)-2.5(3.1)+6.9(-4.3) \\ -3.5-7.75+6.9(-4.3) \end{gathered}[/tex]

Then, we need to solve the multiplication of the last term:

[tex]-3.5-7.75-29.67[/tex]

Now, we can factorize the sign minus, but we will need to change the signs of all terms:

[tex]-(3.5+7.75+29.67)[/tex]

Finally, we can add the term to get:

[tex]\begin{gathered} -(40.92) \\ -40.92 \end{gathered}[/tex]

Therefore, the errors made by student P were at step 1, when this student applies the distributive property, he or she made a mistake with the sign of 7.75. This number should be negative. The next steps are correct by taking into account that the error was in the first step, the result will be wrong.

For student Q, we have that everything was correct until step 3, where he or she factorize the minus sign but he or she doesn't change the signs of all signs. After that, the procedure is correct but the answer will be wrong due to the mistake made at step 3.

a waffle cone with a height of 6 inches has a volume of 56.52 cubic inches. What's the area

Answers

Answer:

28.26 square inches.

Explanation:

Given a waffle cone with the following properties:

• Height = 6 inches

,

• Volume = 56.52 cubic inches.

[tex]\text{Volume of a cone}=\frac{1}{3}\pi r^2h[/tex]

Note that the base of the cone is a circle and the area of a circle:

[tex]A=\pi r^2[/tex]

Substitute the given values:

[tex]\begin{gathered} 56.52=\frac{1}{3}\times\pi\times r^2\times6 \\ 56.52=2\pi r^2 \\ \pi r^2=\frac{56.52}{2} \\ \pi r^2=28.26in^2 \end{gathered}[/tex]

The area​ of the base is 28.26 square inches.

If f(x) = 1 x - 2 v
x-2, what is f¹(x)?

Answers

Answer:

f¯¹(x) =9x+18, for the pictorial(image) question

what is the send question is it asking derivative or inverse

If it is inverse f¯¹(x)=-x-2 as it is

Or if it is derivative f'(x)=-1

Step-by-step explanation:

For the image question f(x)=1/9x-2,f¯¹(x)=?

f(x)=1/9x-2............given

y=1/9x-2................swapping f(x) by y to Easily write

x=1/9y-2................interchanging x and y

1/9y-2=x................changeling side of equation

9(1/9y-2)=(x)9.......multiplying both sides by 9 to override the fraction on the right side

y-18=9x

y=9x-18.................Return to where it were

f¯¹(x)=9x+18..........swap back f¯¹(x) in the y

For the question f(x)=-x-2,f¯¹(x)=?

following the ☝️ arrangement

y=-x-2

x=-y-2

-y-2=x

-y=x+2

y=-x-2

f¯¹(x)=-x-2

Find the additive inverse. −31

Answers

Answer:

To get the additive inverse of a positive number you put a minus in front of it and to get the additive inverse of a negative number, you remove the minus to make it a positive number.

Please help me, I am happy to contribute and learn .

Answers

Explanation:

We are told that the rate at which the pump dumps the pollutant per day to be

[tex]\frac{\sqrt{t}}{15}[/tex]

To solve the question, let us assume that t is the number of days

So, to find the amount dumped after 3 days, we will put t =3 into the equation

[tex]\frac{\sqrt{3}}{15}=\frac{1.732}{15}=0.11547[/tex]

Therefore, the answer is 0.115

You paid $600 for a new guitar. Your guitar cost $40 more than twice the cost of your friends guitar. Wright an equation based on this information.

Answers

Answer:  600 divided by 2 + 40

Step-by-step explanation:

Answer:

2x + 40 = 600

2x = 560

x = 280.

Your friends guitar costs $280.

Step-by-step explanation:

Set the equation equal to $600, the cost of the new guitar. Let the variable, x, represent the cost of your friends guitar. Since your guitar was + 40 more than double the cost of your friend’s, this can be written as an equation:

2x + 40 = 600

What is the x-intercept?

Answers

Answer:2.5

Step-by-step explanation:

        l

        I

------------------ x intersept

        I

        I

y intersept

Answer: 2.5 have a good day

the graph shows the mass of the bucket containing liquid depends on the volume of liquid in the bucket. Use the graph to find the range of the function.

Answers

The graph's range is 0 to M plus or minus 5.5.

Where can I find the function's range?

The set of graph output values that make up a function's range.

This means that the set of y values in the graph is the range of a function.

How do you figure out the domain and range?

The domain

We can observe the following on the function's graph:

The x values range from zero to seven and a half.

This indicates that the domain is 0=x=7.5.

The range

We can observe the following on the function's graph:

Beginning at 0, the x values go all the way up to 5.5.

In other words, the range is 0 = M = 5.5.

To know more about domain and range visit:-

https://brainly.com/question/28135761

#SPJ13

I need help with unit rate fractions pls try to explain very very easily and well and answer quickly i gave an example

Answers

To find out the unit rate

Divide cups of sugar by the teaspoon of vanilla

so

[tex]\frac{2}{3}\colon2=\frac{2}{3*2}=\frac{1}{3}[/tex]

The answer is 1/3

Option A

If ( a + 3 , b – 1 ) = ( - 2 , 4 ) , then a + b =

Answers

Answer: {(1,3),(1,4),(2,3),(2,4)}

Step-by-step explanation:

Step -1: Define the Cartesian product.

              Cartesian product: If A and B are two non empty sets, then  

              Cartesian product A×B is set of all ordered pairs (a,b) such that a∈A and b∈B.

Step -2: Find the Cartesian product of given sets.

              We have given,

              A={1,2} and B={3,4}

              So, A×B={(1,3),(1,4),(2,3),(2,4)}

Hence, option A. {(1,3),(1,4),(2,3),(2,4)} is correct answer.

Make three problem about finding DOMAIN X-intercept Y-intercept Vertical Asymptote Horizontal asymptote

Answers

A graph's domain, which is defined as the entire set of input values visible on the x-axis, refers to the set of possible input values. The possible output values are displayed on the y-axis and make up the range.

What is Vertical and Horizontal asymptote?

Asymptotes are a distinctive feature of the graphs of rational functions. When a curve is nearing the edges of a coordinate plane, it is said to be asymptote. A rational function's vertical asymptotes happen as its denominator gets closer to zero.

In order to cross a vertical asymptote, a rational function must divide by one, which is impossible. When the x-values increase significantly in size, either positively or negatively, horizontal asymptotes develop. You can pass through horizontal asymptotes.

A vertical asymptote of a graph is a vertical line with the equation x = a, where the graph tends toward positive or negative infinity as the inputs get closer to a.

A graph's horizontal asymptote is a horizontal line, y = b, where the graph moves toward the line as the inputs move toward ∞+ or ∞-.

Three problem about finding DOMAIN, X-intercept, Y-intercept, Vertical Asymptote, Horizontal asymptote

1) Determine the vertical asymptote(s), horizontal or slant asymptote, x-intercept(s), y-intercept, and domain. Then, sketch a graph of the function on the given set of axes. Label all asymptotes and intercepts.

[tex]m(x) = \frac{3x^2 -12}{x^2 -7x + 6}[/tex]

2) Determine the Domain, Y-intercept, x-intercept(s), Vertical Asymptote(s), and Horizontal Asymptote, if the exist: Include the multiplicity of the x-intercepts if the multiplicity is greater than 1. Then graph the ratio function.

[tex]v(x) = \frac{3x - 1}{x^2+5x +6}[/tex]

3) What are the Domain, x-intercept, y-intercept, vertical asymptote and horizontal asymptote of the rational function [tex](x^3-x+12/x^2-3x-4)[/tex]?

Learn more about Horizontal asymptote

https://brainly.com/question/4138300

#SPJ9

A new heating and aip constitioner will cost the Benguin fomily $4122,theymake a down payment of 20 percent and finance the remaining amount theyObtain an instaliment loan for 36 months at an APR of 9%A What is the down payment?B What is the amount of the loan?

Answers

The cost of the new heating and air conditioner equipment is:

A = $4122

They make a down payment of 20%

A. The down payment is:

[tex]\begin{gathered} DP=\$4122\times\frac{20}{100} \\ \\ DP=\$824.40 \end{gathered}[/tex]

B The amount of the loan is the remaining amount after paying the down payment:

L = $4122 - $824.40

L = $3297.60

the formula for the volume of a cylinder is V=πr²h A cylinder has a volume of 300p feet³ and a radius of 5 feet (A) Solve the formula V= πr²h for h (B) Find the height of the cylinder

Answers

[tex]V=\pi\cdot r^2\cdot h[/tex]

A) To solve the formula for h:

1. Divide both sides of the equation into π*r²:

[tex]\begin{gathered} \frac{V}{\pi\cdot r^2}=\frac{\pi\cdot r^2\cdot h}{\pi\cdot r^2} \\ \\ \frac{V}{\pi\cdot r^2}=h \end{gathered}[/tex]

B) You have the next data:

V=300πfeet³

r=5feet

Substitute those values in the formula you get in A) and calculate the h:

[tex]\begin{gathered} h=\frac{300\pi\cdot ft^3}{\pi\cdot(5ft)^2} \\ \\ h=\frac{300ft^3}{25ft^2}=12ft \end{gathered}[/tex]Then, the height of the cylinder is 12 feet

The diameter of Jupiter is about 1.43•10^5km. The diameter of the Earth is about 12,700km. About how many times greater is the diameter of Jupiter that the diameter of Earth

Answers

The diameter of the Earth is 11.3 times less than the diameter of the Jupiter

Ratio and proportions

Fractions are written as a ratio of two integers. Given the following parameters;

Diameter of Jupiter = 1.43•10^5km

Diameter of Earth = 1.27 * 10^4km

Find the ratio

Ratio = Jupiter/Earth

Ratio = 1.43•10^5/1.27*10^4

Ratio = 1.13 * 10^1

Ratio = 11.3

Jupiter = 11.3 of Earth

This shows that the diameter of Jupiter if 11.3 times greater than Earth.

Learn more on ratio and proportion here: https://brainly.com/question/12024093

#SPJ1

You roll a 6-sided die two times.What is the probability of rolling a 6 and then rolling a number less than 2?Simplify your answer and write it as a fraction or whole numb

Answers

We are asked to determine the probability of rolling a 6 and then rolling a number less than 2. To do that we will use the product rule probabilities since we want to find the probability of two independent events happening:

[tex]P(AandB)=P(A)P(B)[/tex]

Where:

[tex]\begin{gathered} A=\text{ rolling a 6} \\ B=\text{ rolling a number less than 2} \end{gathered}[/tex]

To determine the probability of rolling a 6 we need to have into account that there are 6 possible outcomes out of which only one is a 6. Therefore, the probability is:

[tex]P(A)=\frac{1}{6}[/tex]

To determine the probability of B we need to have into account that in a 6-sided die the numbers that are less than 2 are (1), this means that there is only one number less than 2 out of 6 possible numbers. Therefore, the probability is:

[tex]P(B)=\frac{1}{6}[/tex]

Now, we substitute in the product rule:

[tex]P(AandB)=(\frac{1}{6})(\frac{1}{6})[/tex]

Solving the product:

[tex]P(AandB)=\frac{1}{36}[/tex]

Therefore, the probability is 1/36.

complex vector question.A bolt is tightened by applying a force to one end of a wrench.

Answers

The Scalar and Cross Product of Vectors

Given two vectors:

[tex]\begin{gathered} \underline{r_1}=(a,b,c) \\ \underline{r_2}=(d,e,f) \end{gathered}[/tex]

The scalar product is defined as:

[tex]\underline{r_1}\cdot\underline{r_2}=ad+be+cf[/tex]

The cross product is the result of computing the following determinant:

[tex]\underline{r_1}\times\underline{r_2}=\begin{bmatrix}i & j & {k} \\ {a} & {b} & {c} \\ {d} & {e} & {f}\end{bmatrix}[/tex]

Where i, j, and k are the unit vectors in each of the directions x, y, and z, respectively.

This concept will be applied to the following physics problem.

Given a force F= (2, 3, 0) and the distance vector d = (4, 0, 0), the torque is defined by:

[tex]\tau=r\times F[/tex]

Calculating:

[tex]\tau=(4,0,0)\times(2,3,0)[/tex][tex]\tau=\begin{bmatrix}{i} & {j} & {k} \\ {4} & {0} & {0} \\ {2} & {3} & {0}\end{bmatrix}[/tex]

Calculating the determinant:

[tex]\begin{gathered} \tau=0i+12k+0j-(0k+0j+0i) \\ \tau=0i+0j+12k \end{gathered}[/tex]

Expressing in vector form τ = (0, 0, 12) <= should use angle brackets

The magnitude of the torque is:

[tex]\begin{gathered} |\tau|=\sqrt[]{0^2+0^2+12^2} \\ |\tau|=\sqrt[]{144} \\ |\tau|=12 \end{gathered}[/tex]

The power P is equal to the scalar product of the torque by the angular velocity w. We are given the angular velocity w = (3, 3, 2), thus:

[tex]\begin{gathered} P=(0,0,12)\cdot(3,3,2) \\ P=0\cdot3+0\cdot3+12\cdot2 \\ P=24 \end{gathered}[/tex]

P = 24

Analyze the diagram. Which quadrilateral is a kite?

Quadrilateral N M O P is shown. Sides P N and N M are congruent.

Quadrilateral A B C D is shown. Sides A D and D C are congruent. Sides A B and B C are congruent.

Quadrilateral N M O P is shown. All sides are different lengths.

Answers

Answer: in the picture

Answer:

Quadrilateral ABCD

Step-by-step explanation:

3) Find the equation of the line:
a) with a gradient of 2 and cutting the y-axis at 7
b) with a gradient of -2 and passing through the point (2;4)
c) passing through the points (2; 3) and (-1; 2)
d) parallel to the x-axis cutting the y-axis at 5

Answers

Step-by-step explanation:

this is very much doing the exact same things as the previous question, just with a little bit different numbers.

remember, gradient = slope.

the slope is always the factor of x in the slope-intercept form

y = ax + b

our in the point-slope form

y - y1 = a(x - x1)

"a" is the slope, b is the y-intercept (the y- value when x = 0).

(x1, y1) is a point on the line.

the slope is the ratio (y coordinate change / x coordinate change) when going from one point on the line to another.

a)

y = 2x + 7

b)

y - 4 = -2(x - 2) = -2x + 4

y = -2x + 8

c)

going from (2, 3) to (-1, 2)

x changes by -3 (from 2 to -1)

y charges by -1 (from 3 to 2)

the slope is -1/-3 = 1/3

we use one of the points, e.g. (2, 3)

y - 3 = (1/3)×(x - 2) = x/3 - 2/3

y = x/3 - 2/3 + 3 = x/3 - 2/3 + 9/3 = x/3 + 7/3

d)

y = 5

this is a horizontal line (parallel to the x-axis) and represents every point on the grid, for which y = 5.

the slope is 0/x = 0, as y never changes at all.

the y- intercept is 5, of course.

Answer:

[tex]\textsf{a) \quad $y=2x+7$}[/tex]

[tex]\textsf{b) \quad $y=-2x+8$}[/tex]

[tex]\textsf{c) \quad $y=\dfrac{1}{3}x+\dfrac{7}{3}$}[/tex]

[tex]\textsf{d) \quad $y=5$}[/tex]

Step-by-step explanation:

Part (a)

Slope-intercept form of a linear equation:

[tex]y=mx+b[/tex]

where:

m is the slope.b is the y-intercept.

Given values:

Slope = 2y-intercept = 7

Substitute the given values into the formula to create the equation of the line:

[tex]\implies y=2x+7[/tex]

---------------------------------------------------------------------------

Part (b)

Point-slope form of a linear equation:  

[tex]y-y_1=m(x-x_1)[/tex]

where:

m is the slope.(x₁, y₁) is a point on the line.

Given:

Slope = -2(x₁, y₁) = (2, 4)

Substitute the given values into the formula to create the equation of the line:

[tex]\implies y-4=-2(x-2)[/tex]

[tex]\implies y-4=-2x+4[/tex]

[tex]\implies y=-2x+8[/tex]

---------------------------------------------------------------------------

Part (c)

Slope formula:

[tex]\textsf{slope}\:(m)=\dfrac{y_2-y_1}{x_2-x_1}[/tex]

where (x₁, y₁) and (x₂, y₂) are points on the line.

Given points:

(x₁, y₁) = (2, 3)(x₂, y₂) = (-1, 2)

Substitute the points into the slope formula to calculate the slope of the line:

[tex]\implies m=\dfrac{2-3}{-1-2}=\dfrac{-1}{-3}=\dfrac{1}{3}[/tex]

Substitute the found slope and one of the points into the point-slope formula to create the equation of the line:

[tex]\implies y-y_1=m(x-x_1)[/tex]

[tex]\implies y-3=\dfrac{1}{3}(x-2)[/tex]

[tex]\implies y-3=\dfrac{1}{3}x-\dfrac{2}{3}[/tex]

[tex]\implies y=\dfrac{1}{3}x+\dfrac{7}{3}[/tex]

---------------------------------------------------------------------------

Part (d)

Slope-intercept form of a linear equation:

[tex]y=mx+b[/tex]

where:

m is the slope.b is the y-intercept.

If the line is parallel to the x-axis, its slope is zero.

If the line intersects the y-axis at y = 5, then its y-intercept is 5.

Therefore:

m = 0b = 5

Substitute the given values into the formula to create the equation of the line:

[tex]\implies y=0x + 5[/tex]

[tex]\implies y=5[/tex]

I need help with 18 I need an answer and a explanation

Answers

Mark's height last year was 46 inches.

Mark definitely grows over the past year. Let the height he grew be x.

Then, Mark's new height will be

[tex](46+x)\text{ inches}[/tex]

Let us represent Mark's height with M and Peter's height with P.

This means that

[tex]M=46+x\text{ -----------(a)}[/tex]

and, from the question, Peter's height is

[tex]P=51\text{ ----------(b)}[/tex]

The question says that Mark's height is 3 inches less than Peter's height. This we can write as

[tex]M=P-3\text{ -------------(c)}[/tex]

Therefore, if we put Mark's and Peter's ages into equation c, we can find a value for x as follows:

[tex]\begin{gathered} 46+x=51-3 \\ 46+x=48 \end{gathered}[/tex]

Since 46 + x = M, then Mark's height is 48 inches

List the factors to find the GCF of 24 and 12

Answers

Given:

GCF of 24 and 12.

[tex]\begin{gathered} 24=2^3\times3 \\ 12=2^2\times3 \end{gathered}[/tex][tex]\begin{gathered} \text{GCF of 24 and 12=}3\times2^2 \\ \text{GCF of 24 and 12=}12 \end{gathered}[/tex]


A price p (in dollars) and demand x (in items) for a product are related by 2x²-5xp + 55p²-23,200.
If the price is increasing at a rate of 3 dollars per month when the price is 20 dollars, find the rate of change of the demand with respect to time. (Round your answer to four
decimal places.)

Answers

The monthly rate of change in demand is -$40.7.

How is the rate of change estimated from an equation?

The slope of a graphed function is determined using the average rate of change formula. The method for finding the slope is differentiation.

A price-demand relation equation is given.

2x²-5xp + 55p²=23,200.

Differentiate the given equation with time

[tex]\begin{aligned}&4 x \frac{d x}{d t}-5\left(x \frac{d p}{d t}+p \frac{d x}{d t}\right)+110 p \frac{d p}{d t}=0 \\&4 x \frac{d x}{d t}-5 p \frac{d x}{d t}=5 x \frac{d p}{d t}-110 p \frac{d p}{d t} \\&(4 x-5 p) \frac{d x}{d t}=(5 x-110 p) \frac{d p}{d t} \\&\frac{d x}{d t}=\frac{(5x-110 p)}{(4 x-5p)} \frac{d p}{d t}\end{aligned}[/tex]

Put the value of p in the original equation.

For p=20

[tex]2x^{2} -5x\times 20+ 55\times20^{2}=23200\\2x^{2}-100x+22000=23200\\2x^{2}-100x-1200=0\\x^{2}-50x-600=0\\x=60 \text{ or }-10[/tex]

Since the price can not be negative, x=60.

Putting these values in the differential equation.

[tex]\frac{d x}{d t}=\frac{(5 x-110 p)}{(4 x-5 p)} \frac{d p}{d t}\\=\frac{(5\times60-110\times 20)}{(4\times60-5 \times20)} \times3\\=\frac{300-2200}{140}\times3\\ =-40.7[/tex]

So, the monthly rate of change in demand is -$40.7.

The minus sign indicates that demand is decreasing.

To know more about the rate of change:

https://brainly.com/question/19752248

#SPJ9

1. Last year the price of a college textbook(b) was $197. This year the price will be 13% higher. Which expression shows the difference in price from last year to this year? 1. b * 0.13 B. b.1.13 C.b-0.13 D.b - 13

Answers

B) b*1.13

1) Since the College textbook's price has been raised up 13%, then we can write:

b(1 +0.13)

b(1.13)

Plugging b = $197 we have:

197(1 +0.13)

197 (1.13)

$222.61

2) So the factor that expresses that difference (13% up) is 1.13

For 1 is equivalent to 100% and 0.13 to 13%

3) Hence, the answer is b* 1.13 calling b that $197.

Name two rays that contain the following line segments:• BC• GH

Answers

Two rays that contain the given line segment BC is  [tex]\overrightarrow {EC}[/tex]  and line segment GH is [tex]\overrightarrow {EH}[/tex] .

The length of a line segment is its measurement. Unlike a line that extends continuously, a line segment has a set length and is easy to measure.

The next link in the chain is Ray. It is made up of a line and even a mix of line segments with one terminating end and an eternally extending end.

Due to one of its ends not terminating, its length cannot be determined. Line segments are parts of a line that have two endpoints.

The construction of various shapes, such as triangles, polygons, hexagons, and squares, involves the use of a number of line segments.

From the diagram we can see that the rays EC and EH contains the given line segments. From the Rays the other line segments are BD, CD , GH.

AH is another ray.

Two rays that contain the given line segment BC is  [tex]\overrightarrow {EC}[/tex]  and line segment GH is [tex]\overrightarrow {EH}[/tex] .

To learn more about line segment visit:

https://brainly.com/question/1534220

#SPJ9

Apollo Enterprises has been awarded an insurance settlement of $6,000 at the end of each 6 month period for the next 12 years. calculate how much (in $) the insurance company must set aside now at 6% interest compounded semiannually to pay this obligation to Apollo

Answers

$12180 the insurance company must set aside now at 6% interest compounded semiannually to pay this obligation to Apollo.

This is a problem from the compound interest system. We can solve this problem by following a few steps.

Apollo Enterprises has been awarded an insurance settlement of $6,000 at the end of each 6-month period for the next 12 years with a 6% interest rate. We have to calculate the total amount after 12 years.

To solve this problem we should know the formula for the compound interest method.

Formula:-

A = P {(1 + r/n)^(n.t)}

Here,

A denotes the final amount, we have to find this.P denotes the initial principal balance which is $6,000r denotes the interest rate which is 6%n denotes the number of times interest is applied per time period which is 12/6 = 2. t denotes the number of time periods elapsed which is 12 years.

Now, we can calculate the value of A.

A = 6000 {( 1 + 6/200 )^2.12} = 6000 ( 1 + 6/200 )^24 = 6000 × 2.03 = 12180

Therefore, the total amount after 12 years is $12180

         

To learn more about compound interest visit,

https://brainly.com/question/26457073?referrer=searchResults

#SPJ9

answer f 1 half 25 y intercept equals 375--g slope 1 half 25 y intercept equal 15H slope equals 25 y intercept equal 375J slope equals negative 25 y intercept equals 15

Answers

Answer:

[tex]\begin{gathered} \text{Slope}=-\frac{1}{25} \\ y-\text{intercept}=15 \end{gathered}[/tex]

Step-by-step explanation:

Linear functions are represented by the following expression:

[tex]\begin{gathered} y=mx+b \\ \text{where,} \\ m=\text{slope} \\ b=y-\text{intercept} \end{gathered}[/tex]

m is the constant rate of change of the function, and it's calculated as the change in y over the change in x:

[tex]\begin{gathered} m=\frac{y_2-y_1}{x_2-x_1} \\ m=\frac{14.6-15}{10-0} \\ m=-\frac{1}{25} \end{gathered}[/tex]

The y-intercept of a linear function is when the line crosses the y-axis, which means when x=0.

Therefore, the y-intercept of the line is 15.

Other Questions
HELP ASAP ASAPRead the following passage from Anne of Green Gables by Lucy Maud Montgomery. Answer the question that follows the text."Oh, you can talk as much as you like. I don't mind.""Oh, I'm so glad. I know you and I are going to get along together fine. It's such a relief to talk when one wants to and not be told that children should be seen and not heard. I've had that said to me a million times if I have once. And people laugh at me because I use big words. But if you have big ideas you have to use big words to express them, haven't you?""Well now, that seems reasonable," said Matthew."Mrs. Spencer said that my tongue must be hung in the middle. But it isn'tit's firmly fastened at one end. Mrs. Spencer said your place was named Green Gables. I asked her all about it. And she said there were trees all around it. I was gladder than ever. I just love trees. And there weren't any at all about the asylum, only a few poor weeny-teeny things out in front with little whitewashed cagey things about them. They just looked like orphans themselves, those trees did. It used to make me want to cry to look at them. I used to say to them, 'Oh, you POOR little things! If you were out in a great big woods with other trees all around you and little mosses and Junebells growing over your roots and a brook not far away and birds singing in you branches, you could grow, couldn't you? But you can't where you are. I know just exactly how you feel, little trees.' I felt sorry to leave them behind this morning. You do get so attached to things like that, don't you? Is there a brook anywhere near Green Gables? I forgot to ask Mrs. Spencer that.""Well now, yes, there's one right below the house.""Fancy. It's always been one of my dreams to live near a brook. I never expected I would, though. Dreams don't often come true, do they? Wouldn't it be nice if they did? But just now I feel pretty nearly perfectly happy. I can't feel exactly perfectly happy becausewell, what color would you call this?"She twitched one of her long glossy braids over her thin shoulder and held it up before Matthew's eyes. Matthew was not used to deciding on the tints of ladies' tresses, but in this case there couldn't be much doubt."It's red, ain't it?" he said.The girl let the braid drop back with a sigh that seemed to come from her very toes and to exhale forth all the sorrows of the ages."Yes, it's red," she said resignedly. "Now you see why I can't be perfectly happy. Nobody could who has red hair. I don't mind the other things so muchthe freckles and the green eyes and my skinniness. I can imagine them away. I can imagine that I have a beautiful rose-leaf complexion and lovely starry violet eyes. But I CANNOT imagine that red hair away. I do my best. I think to myself, 'Now my hair is a glorious black, black as the raven's wing.' But all the time I KNOW it is just plain red and it breaks my heart. It will be my lifelong sorrow. I read of a girl once in a novel who had a lifelong sorrow but it wasn't red hair. Her hair was pure gold rippling back from her alabaster brow. What is an alabaster brow? I never could find out. Can you tell me?""Well now, I'm afraid I can't," said Matthew, who was getting a little dizzy. He felt as he had once felt in his rash youth when another boy had enticed him on the merry-go-round at a picnic."Well, whatever it was it must have been something nice because she was divinely beautiful. Have you ever imagined what it must feel like to be divinely beautiful?""Well now, no, I haven't," confessed Matthew ingenuously."I have, often. Which would you rather be if you had the choice-divinely beautiful or dazzlingly clever or angelically good?""Well now, I-I don't know exactly.""Neither do I. I can never decide. But it doesn't make much real difference for it isn't likely I'll ever be either."In a minimum of one paragraph, explain what you learn about these characters from this interaction. Use details from the passage to support your response. Following the invasion of Algeria in 1830, what did France bring to their African colonies? How did US pilots contribute to the Allied war effort in Europe during World War II?0 They carried out countless bombing raids on targets in Germany.O They sank ships carrying supplies to Britain and the Soviet Union.They trained British pilots in effectiveair support techniques.They ferried retreating troops between Europe and Great Britain. The manager of a new restaurant plans on ordering place-mats for the maximum number of diners, which is 279. Suppose the place-mats come in boxes of 24. Write a division expression that could be used to determine the number of boxes he needs to order.\ .question 9a data analyst is using statistical measures to get a better understanding of their data. what function can they use to determine how strongly related are two of the variables? PLEASE HELP !! When a right triangle with a hypotenuse of 1 is placed in the unit circle, which sides of the triangle correspond to the x- and y-coordinates? Tricia buys 18 apples for $27. The total cost, c, of the apples is proportional to the number of apples, a, that she buys. How much does one apple cost? Create an equation using c and a to represent the proportional relationship. convert 4520 into second supercomputers are machines that could compute quadrillions of floating-point operations per second. that is, their performance is multiples of 10 15 1015 operations per second. these machines are used for a variety of things, including simulating earthquakes, nuclear reactions, and the big bang. in fact, supercomputer simulations have at this point replaced the testing of actual nuclear arms. some of the advantages of these simulations include: select two answers. elimination tool select all that apply a models can incorporate all of the complexities of nuclear reactions. b models are cheaper than building multiple designs and testing each in the real world. c modern supercomputers are cheaper than a mid-size car. d simulations come without the environmental risks associated with nuclear tests. eleven-year-old tommy eats large amounts of food and finds eating very enjoyable. he seems to have very little control over his appetite. his doctor suspects that tommy has a shortage of the hormone: A gambling game at a local carnival is played as follows: each time someone plays the game, he or she pays $3. A fair die is rolled, and if the die has an even roll (e.g. 2, 4, or 6), he or she wins nothing. If it is an odd roll, he or she wins the amount of money (in dollars on the face of the diee.g. if he or she rolls a 1, he or she wins $1; a 3, he or she wins $3; a 5, he or she wins $5). The random variable [tex]X[/tex] represents the amount he or she wins per play of the game, not including how much he or she pays.Part 1: Complete the given table.Part 2: How much can someone expect to win on average per play of the game? Should he or she pay $3 to play this game? Why or why not? Using the distance formula, d = (x2 - x1)2 + (y2 - y1)2, what is the distance between point (0, 5) and point (3, -1) rounded to the nearest tenth? fitzgerald oil and gas incurred costs of $9.24 million for the acquisition and development of a natural gas deposit. the company expects to extract 4.00 million cubic feet of natural gas during a four-year period. natural gas extracted during years 1 and 2 were 800,000 and 900,000 cubic feet, respectively. what was the depletion for year 1 and year 2? Conjugate, pleaseThanks in advance!!l ___ (pedir)Nosotros ___ (pedir)Ellas ___ (creer)Ustedes ___ (poder)Yo ___ (creer) a soft drink company holds a contest in which a prize may be revealed on the inside of the bottle cap. the probability that each bottle cap reveals a prize is 0.2 and winning is independent from one bottle to the next. what is the probability that a customer loses on their first two bottles and wins on their third bottle? Which parenting style is most closely associated with children who are inclined to be withdrawn and tend to do poorly in school?Question 5 options:Permissive parentsInconsistent parentsAuthoritarian parentsAuthoritative parents These two equations look very similar at first. What is the difference in how you would solve them? `\frac{x-2}{3}=5` `\frac{x}{3}-2=5` why is 2 a factor of all even numbers Describe how the setting shapes the character Salva in chapter 5? from long walk to water list 2 details and explain. PLEASE HELLPP ME YOO Be able to discuss some solutions or strategies to reduce the impact of the above pollutants on ecosystems.